#14 A scale drawing of a rectangular room measures 2 cm by 4 and 1/4 cm. E
The scale used for the scale drawing is 1/2 cm = 5 ft. Carpeting will cost
$8.50 a square foot. How much will it cost to buy enough carpet to cover
the floor of this rectangular room? (**numbers and decimal only in answer
** NO $$ NO comma**) *

Answers

Answer 1

Answer:

Step-by-step explanation:

Width

1/2 cm = 5 feet

2cm = x                    Cross Multiply

1/2 x = 2 * 5              Combine

1/2 x = 10                  Multiply both sides by 2

x = 20

Length

1/2 cm = 5 feet

4 1/4 cm = x              Cross Multiply

1/2 x = 4 1/4 * 5         Combine the right

1/2 x = 21.25              Multiply both sides by 2

x = 42.5                    

Room Area

Area = 20 * 42.5

Area = 850 square feet

Cost

1 square foot costs 8.5 dollars.

850 square feet costs 8.5 * 850

850 square feet costs 7225  dollars.


Related Questions

simplify 4 whole 2/7 + 5/7

Answers

Answer:

1

Step-by-step explanation:

2/7+5/7=7/7=1 because 7/7 would be a 1 whole

Answer:

35/7

Step-by-step explanation:

4 = 28/7

28/7 + 2/7 + 5/7 = 35/7

HEHEHEHE HELP LOL IM NOT SMART ENOUGH FOR THIS 3

Answers

a is an absolute value if it is greater than zero. This simply means that an absolute value number will always be greater than zero. This is because absolute value is the distance from 0 on a number line. Keep in mind that distance cannot be negative. For example, look at the numbers 5 and -5. 5 is 5 spaces from 0, and -5 is also 5 spaces from 0, not -5 spaces from 0. Put simply, an absolute value number that is greater than 0, will always be positive.

Feel free to mark Brainliest!

Find the area and the circumference of a circle with diameter 7 cm.
Use the value 3.14 for it, and do not round your answers. Be sure to include the correct units in your answers.
cm
Area:
7 cm
Circumference:
Х

Answers

Answer:

area = 38.465^2 cm

circumference = 21.98 cm

Step-by-step explanation:

area =

divide the diameter by itself and multiply radius by itself and multiply by 3.14

7/2 = 3.5 | (3.5)^2 = 12.25 | (12.25) (31.4) = 38.465^2 cm

circumference =

multiply the diameter by 3.14

(7) (3.14) | 21.98

^2 (squared)

Mathew basketball team won 68% of the games they played if they won 17 games how many did they play

Answers

Answer:

25

Step-by-step explanation:

turn 68% into a decimal (or keep as a percentage, your choice) and divide

17 divided by 0.68 (68% as a decimal) = 25

The question in the image! Please hurry :D

Answers

Answer:

8.85 * 10 ^17

Step-by-step explanation:

8.2 * 10 ^17 + 6.5 * 10 ^16

They must be to the same power to add

8.2 * 10 ^17 + .65 * 10 ^17

Add

8.85 * 10 ^17

help asap whoever right get brainiest

Answers

Answer:

EF

Step-by-step explanation:

I think the answer is EF because turning the line counterclockwise and 80 degree angle would be that.

Write the expression as the sum or difference of two logarithmic functions containing no exponents.
log(x/x + 1)

Answers

Hi how u doin ?

[tex] log( \frac{x}{x + 1} ) = \\ [/tex]

[tex] log(x) - log(x + 1) [/tex]

Have a great day ❤

ILL MARK BRAINLIEST HELP PLS

Answers

Answer:

Step 1, do math:

-13.6  >= 1.7x

-8 >= x

answer: C

The answer should be the first one!

Determine f –1(x) if f of x is equal to the quantity x plus 3 end quantity over the quantity x minus 4 end quantity where x ­ 4.
f inverse of x is equal to the quantity x minus 1 end quantity over the quantity 4 times x plus 3 end quantity
f inverse of x is equal to the quantity 4 times x plus 3 end quantity over the quantity x minus 1 end quantity
f inverse of x is equal to the quantity x minus 4 end quantity over the quantity x plus 3 end quantity
f inverse of x is equal to the quantity x minus 3 end quantity over the quantity x plus 4 end quantity

Answers

Answer:

Option B

Step-by-step explanation:

Given function:

f(x) = (x + 3)/(x - 4)

Find its inverse, take f(x) as y and swap x with y

x = (y + 3)/(y - 4)x = (y - 4 + 7) / (y - 4)x = 1 + 7/(y - 4)x - 1 = 7 / (y - 4)y - 4 = 7 / (x - 1)y = 4 + 7 / (x - 1)y = (4x - 4 + 7)/ (x - 1)y = (4x + 3) / (x - 1)

Replace y with f⁻¹(x):

f⁻¹(x) = (4x + 3) / (x - 1)

Correct choice is B

A x is equal to the quantity x plus 3 end quantity over the quantity x minus 4 end quantity where x ­ 4. at x = 0.5, the inverse function f²(-1)(x)  f²(-1)(x) = 0.5

To determine the inverse of the function f(x) = (x + 3) / (x - 4),  to follow these steps:

Replace f(x) with -1, since you're looking for the inverse at the point where f(x) = -1:

-1 = (x + 3) / (x - 4)

Solve for x in terms of -1:

Multiply both sides by (x - 4):

-1 ×(x - 4) = x + 3

Distribute the -1:

-x + 4 = x + 3

Solve for x:

Subtract x from both sides:

4 = 2x + 3

Subtract 3 from both sides:

1 = 2x

Divide by 2:

x = 0.5

To know more about function here

https://brainly.com/question/30721594

#SPJ3

Need help with this 3 questions

Answers

Answer:

  68. (-1, 0)

  69. (0, 2)

  70. (2, -1)

Step-by-step explanation:

The solution is the ordered pair (x, y) where the lines intersect. The graphs have no scale, so we must assume 1 grid space corresponds to 1 unit.

The first coordinate of the pair is the horizontal distance from the y-axis. Positive is to the right. The second coordinate of the pair is the vertical distance from the x-axis. Positive is up.

68. The point of intersection is 2 left of the origin on the x-axis. x=-2, y = 0.

  (x, y) = (-2, 0)

__

69. The point of intersection is 2 up from the origin on the y-axis: y = 2, x = 0.

  (x, y) = (0, 2)

__

70. The point of intersection is 2 right and 1 down from the origin: x = 2, y = -1.

  (x, y) = (2, -1)

_____

It is a good idea to learn to read the coordinates of a point from a graph. You will be doing it a lot in your math courses.

Write the equation of the line that passes through the points (8,3) and (7,3). Put your answer in fully reduced point-slope form, unless it is a vertical or horizontal line

Answers

Step-by-step explanation:

the equation :

(y-3)/(3-3) = (x-8) /(7-8)

(y-3) /0 = (x-8) /(-1)

(-1)(y-3) = 0(x-8)

-y +3 = 0

y = 3

(this is a horizontal line)

Answer:

y = 3 (Horizontal line)

Step-by-step explanation:

point: (8, 3) and (7, 3)

(y - y1)/(y2 - y1) = (x - x1)/(x2 - x1)

(y - 3)/(3 - 3) = (x - 8)/(7 - 8)

(y - 3)/0 = (x - 8)/(-1)

(-1)(y - 3) = 0

-y + 3 = 0

-y = - 3

y = 3 (Horizontal line)

Help what is thisss
GIVING OUT BRAINLY IF U ANSWER
idc if sum r wrong

Answers

Answer:

it is a b and c

Step-by-step explanation:

1.
Find the value of 5x? if x = -3.

Answers

Answer:

-15

Step-by-step explanation:

5 * -3 = -15

x = -3

5x = 5(-3)

5(-3) = -15

What is the output of a function when tha input is -9

Answers

Answer: f(-9)

Step-by-step explanation:

A function is described in the form of f(x)

Therefore, if this is how a function is stated we have x as the input.

Here, the x is equal to -9

So now:

f(-9) after substitution.


[tex] \frac{dh}{dt} = - \sqrt{5h} [/tex]

Determine
[tex] \frac{ {d}^{2}h }{d {t}^{2} } [/tex]
at h=16


I have attempted this and keep on getting about -0.28, but that is not an option.​

Answers

If

[tex]\dfrac{dh}{dt} = -\sqrt{5h} = -\sqrt{5} \cdot h^{\frac12}[/tex]

then its derivative, using the power rule, is

[tex]\dfrac{d^2h}{dt^2} = -\sqrt{5} \cdot \dfrac12 h^{1-\frac12} = -\dfrac{\sqrt5}2 h^{-\frac12} = -\dfrac{\sqrt5}{2\sqrt h} = -\sqrt{\dfrac{5}{4h}}[/tex]

At h = 16, the second derivative has a value of

[tex]-\sqrt{\dfrac{5}{4\cdot16}} = -\sqrt{\dfrac{5}{64}} = -\dfrac{\sqrt5}{8} \approx -0.279508[/tex]

Your answer is essentially correct. Either what you submitted doesn't have enough precision or you're expected to give an exact answer (-√5/8).

Find the volume of the following square pyramid.

Answers

Check the picture below.

[tex]\textit{volume of a pyramid}\\ V = \cfrac{Bh}{3}~~ \begin{cases} B = \stackrel{\textit{area of its base}}{50\times 50}\\ h = \stackrel{\textit{its height}}{60} \end{cases}\qquad \implies V = \cfrac{50\cdot 50\cdot 60}{3}\implies \boxed{V= 50000}[/tex]

What is 10/10 as a whole number

Answers

✫ Hello there! ✫

10/10 as a whole number is 1.

If a fraction's numerator is the same as its denominator, then the fraction equals 1.

Hope it helps!

~Just a joyous gal

#CarryOnLearning

Answered by

[tex]-SilentNature-[/tex]

The [tex]\frac{10}{10}[/tex] as the whole number is 1.

Given that,

The fraction is [tex]\frac{10}{10}[/tex]We need to write in a whole number.

Based on the above information, the calculation is as follows:

[tex]= \frac{10}{10}[/tex]

= 1

Learn more:brainly.com/question/17429689

The volume of a gas in cubic centimeters, V, varies inversely with the
pressure of the gas in liters per square centimeter, p.
When the volume of the gas is 16 cm, its pressure is 20 L/cm2. Find the
volume of the gas when the pressure is 2.5 L/cm2
A. 322.5 cm
B. 800 cm3
C. 128 cm3
D. 0.008 cm

Answers

Answer:

C. 128 cm³

Step-by-step explanation:

16(20) = 320

320/2.5 = 128 cm³

Solve for x: 49.98+12x=61.98+7x

Answers

Answer:

X=2.436

Step-by-step explanation:

6= 2(y+2) i need help on this

Answers

Y = 1
This the answer just trust

This population chart represents linear or exponential growth?

•linear
•exponential

Answers

This is Linear graph.

Linear graph is represented in the form of a straight line. ... If the graph of any relation gives a single straight line then it is known as a linear graph. The word "linear" stands for a straight line. The linear graph is a straight line graph that is drawn on a plane connecting the points plotted on x and y coordinates.

- BRAINLIEST answerer

What is the equation of this line?
A. y= x - 3 =
B. y=-x-3 =
C. y= -2x - 3
D. y=2x-3 - - 3​

Answers

do you have a picture of the line ?

PLEASE HELP !!! PICS DOWN BELOW

Answers

Answer:

262.25 $

Step-by-step explanation:

A. Direction: Arrange the given number tiles in a correct order to make it divisible to the given number. Apply the divisibility rules. Given Answer 3 8 4 3 2 9 1 6 8 3 8 3 8 0 5 1. Divisible by 8 2. Divisible by 4 3. Divisible by 11 4. Divisible by 12 5. Divisible by 4 6. Divisible by 8 7. Divisible by 11 8. Divisible by 12 9. Divisible by 4 10. Divisible by 8 7. 2 5 4 5 9 1 0 4 7 6 4 1 6 2 1​

Answers

Sorry i don’t understand its a bit confusing

The speed of a stream is 2 mph. A boat travels 20 miles upstream in the same time it takes to travel 40 miles downstream. What is the speed of the boat in still water?
The speed of the boat in still water is mph.
(Simplify your answer. Type an integer or a decimal.)

Answers

Answer: The speed of a stream is 4 mph. A boat travels 7 miles upstream in the same time it takes to travel 15 miles downstream. What is the speed of the boat in still water…

Step-by-step explanation: The speed of a stream is 4 mph. A boat travels 7 miles upstream in the same time it takes to travel 15 miles downstream. What is the speed of the boat in still water?

Answer: The speed of a stream is 4 mph. A boat travels 7 miles upstream in the same time it takes to travel 15 miles downstream. What is the speed of the boat in still water…

Step-by-step explanation: hope this  helps

HELPP!!! I have a root test for a series but it's in n/2 power, help me out please ?

Answers

The answer is square root of 2/5

Sally can paint a room in 4hours while it takes Steve 3 hours to paint the same room. How long would it take them to paint the room if they worked together.


It would take ___ hours.
(Type an integer of decimal rounded to one decimal place as needed)

Answers

Answer:It would take 1.700 hours.

Step-by-step explanation:

Answer:

Step-by-step explanation:     then it would be 7 or 9 hours

Aaliyah has a loyalty card good for a discount at her local hardware store. The item she wants to buy is priced at $39, before discount and tax. After the discount, and before tax, the price is $33.54. Find the percent discount.

Answers

Answer:

14 % discount

Step-by-step explanation:

Item she wants to buy is priced at $39

After the discount, and before tax, the price is $33.54.

39.00 x .14 = 5.46

39.00 - 5.46 = $33.54

Answer:

16.13

Step-by-step explanation:

Just give me the answer please

Answers

Answer:

what

Step-by-step explanation:

Answer:

Appologies but Why do you have pOn open?

Step-by-step explanation:

umm that is inapropriate but idrc

Julie has 300ml of juice, She gives of the juice away.
How much juice does she give away?

Answers

Answer: Up to 300 ml because that is all she has in total. I think the question is missing parts.

Step-by-step explanation:

Other Questions
Excuse me. (1)__________________________________in this city? Turkey sandwiches cost $2. 50 and veggie wraps cost $3. 50 at a snack stand. Ben has sold no more than $30 worth of turkey sandwiches and veggie wraps in the first hour of business. Let x represent the number of turkey sandwiches and y represent the number of veggie wraps. The inequality 2. 50x+3. 50y30 represents the food sales in the first hour. If ben has sold 4 veggie wraps, what is the maximum number of turkey sandwiches ben could have sold? 5 6 7 10. Whats boiling/melting point Which graph represents the solution to 5 +8x when a force is applied to an object its, ________, the object may ____________________. 8 Qu material utilizaron los artistas de Nok para haceresculturas de humanos?A oro B Hierro C Sal D terracota hey guys will you do a help please mark brain list for user Kim Rosalie WILL GIVE BRAINLIEST AND I SWEAR TO GOD IF ONE MORE BOT ANSWERS AND MAKES IT TO WHERE NO ONE ELSE CAN ANSWEROne challenge of federalism is that power and authority are concentrated in the central government and can take away peoples rights sometimes officials from different levels of government disagree on who should be in charge of a situation local governments have the most power, which creates problems for trade with other nations officials from state governments have too many rules to follow to be effective leaders Un cilindro de 509.8 gramos que tiene un volumen de 1007.1cm, cual es la densidad using continuous division, fill on the correct information in each blank . SOMEONE PLEASE HELP I WILL MARK BRAINLYEST !!Find the unknown side of the triangle below.A. 23.6mB. 37.0m C. 16.9 mD. 28.2m who was the religious leader who led a slave revolt in 1831? Need help with my homework According to the speaker, what two roles does the soulplay?friend and spyfriend and enemyenemy and treasonenemy and sovereignIts Friends and spy How do I solve-3x+y=24-2x=+18+5y Informed consent is involuntary consent on the part of the patient.TrueFalse Solve the equation for the given variable1/2 +y= 2/3A) y=1/6B) y=1/3C) y=1/3D) y=1 1/6 The Necessary and Proper Clause in Article 1, Section 8 of the Constitution gives Congress ___ powers.impliedexpressedex post facto BudgetsTerrance budgets $435 of his salary for car expenses. His salary is $2,900. What percent of his salary isbudgeted for car expenses? Help with this problem pls!